MBE questions wrong 2 Flashcards Preview

Wrong questions > MBE questions wrong 2 > Flashcards

Flashcards in MBE questions wrong 2 Deck (87)
Loading flashcards...
1
Q

Hearsay- prior inconsistent statement to impeach a witness

A

To prove a prior inconsistent statement by extrinsic evidence, certain requirements must first be met. It is not enough that the prior inconsistent statement cast doubt on the witness’s credibility. Instead, the statement must be relevant to the case; i.e., it cannot be a collateral matter. Furthermore, the witness generally must be given an opportunity to explain or deny her statement at some point during trial, and the adverse party must be given an opportunity to examine the witness about the statement. There are certain exceptions to this foundational requirement. For instance, if the inconsistent statement was made by a hearsay declarant, the declarant may be impeached despite the lack of a foundation. Furthermore, the foundation is not required when the inconsistent statement qualifies as an opposing party’s statement

The witness’s prior inconsistent statement is not hearsay if it was made at a deposition, a prior hearing, or to a grand jury. Prior inconsistent statements made under oath at a prior trial or proceeding, or in a deposition, are not hearsay under the Federal Rules if the declarant is now testifying and subject to cross-examination.

Furthermore, a testifying witness’s prior inconsistent statement made while under oath and subject to penalty of perjury in a deposition or prior hearing is not hearsay under the Federal Rules, and is therefore admissible as substantive evidence of the facts asserted. (A) is therefore incorrect.

2
Q

What? - hearsay - prior inconsistent statement?

A

Under Federal Rule 806, the credibility of an unavailable declarant may be attacked by evidence that would be admissible if the declarant had testified as a witness. There is no requirement that a declarant must be present at trial to be impeached. If the declarant is impeached with evidence of her prior inconsistent statement, the foundational requirement that she must explain or deny her statement does not apply. Furthermore, where the declarant’s credibility is impeached, it may also be rehabilitated.

3
Q

The plaintiff, an electrical contractor, sued the defendant homeowner for refusal to pay for extensive wiring repairs performed on his home by the plaintiff’s employee. The plaintiff called the employee to the stand. The employee, under oath, testified that he did not perform any work at the defendant’s home. The employee also denied writing a letter to a friend telling the friend that the employee was going to do electrical work on the home. Without releasing the employee as a witness, the plaintiff offers into evidence the letter written by the employee to his friend.

If the employee’s letter to his friend is properly authenticated, should the court admit the letter?

A

The letter is admissible as substantive evidence as well as for impeachment purposes. For the purpose of impeaching the credibility of a witness, a party may show that the witness has, on another occasion, made statements that are inconsistent with some material part of his present testimony. This may be done by first questioning the witness as to the prior inconsistent statement that he has made. If the witness denies having made the statement or fails to remember it, the making of the statement may be proved by extrinsic evidence. A proper foundation must be laid by giving the witness an opportunity to explain or deny the statement, and it must be relevant to some issue in the case. Here, the plaintiff’s employee has denied that he wrote the letter to his friend. The plaintiff can then impeach the employee by offering the letter into evidence. Because the employee has not been released as a witness, he will have an opportunity to explain or deny the statement, and it is relevant to whether any work was done at the home. Because prior inconsistent statements are generally hearsay, they often are admissible only for purposes of impeachment. The Federal Rules do categorize a testifying witness’s prior inconsistent statement as nonhearsay if it was made under penalty of perjury at a prior trial, hearing, or proceeding, or in a deposition. Here, of course, the employee’s letter to the friend was not made under oath, so it is hearsay.

However, it is still admissible as substantive evidence because it falls within an exception to the hearsay rule. Under Rule 803(3), a statement of a declarant’s then-existing state of mind is admissible as a basis for a circumstantial inference that the declarant acted in accordance with his state of mind. [See also Mutual Life Insurance Co. v. Hillmon (1892)] The employee’s statement that he was going to do electrical work on the home is admissible as circumstantial evidence tending to show that he followed through with his plans and did the electrical work, which is what the statement is being offered to establish.

4
Q

The defendant was being sued for striking and seriously injuring a child with his car one evening while the child was playing in the street near the curb. At trial, the attorney for the child’s parents seeks to have the defendant’s wife testify that he had told her what had happened as soon as he had gotten home, and that he had said, “Between you and me, just before all this happened, I took a quick peek at the back seat to make sure I brought my briefcase home with me. If I had kept my eyes on the road, I never would’ve hit the kid.” The wife was recently divorced from the defendant and eager to testify against him. The attorney for the child’s parents also presented evidence that, unknown to either the defendant or his wife, their neighbor overheard this conversation through her open window.

Assuming a proper objection by the defense attorney, will the wife be permitted to so testify?

A

The defendant’s statement to his wife was made in reliance upon the intimacy of what was at that time their marital relationship. Thus, he has a privilege to prevent her from disclosing the statement. Either spouse (whether or not a party) has a privilege to refuse to disclose, and to prevent another from disclosing, a confidential communication made between the spouses while they were married. Divorce does not terminate this privilege retroactively. At the time that the defendant made the subject statement to his wife, they were married. Given that the statement essentially constituted an admission of liability by the defendant, that he prefaced it with “between you and me,” and that he made the statement in the privacy of their home, it seems likely that the statement was made in confidentiality and in reliance upon the intimacy of the marital relationship. Thus, both the defendant and his wife may refuse to disclose, and may prevent the other from disclosing, the statement. Consequently, the defendant can prevent the wife from testifying to the statement.

5
Q

The police arrested the defendant and charged him with murder. After the defendant’s arrest, two police officers went to his home, where they found his wife. The victim had been killed on the night of March 13, and the officers asked the wife to give them the jacket that the defendant wore on the evening of March 13. Without saying a word, the wife handed the officers a jacket that was covered with bloodstains. Crime lab tests established that the blood on the jacket matched the victim’s blood characteristics. At the defendant’s trial for murder, the prosecution seeks to introduce the jacket into evidence.

Assuming the prosecution successfully establishes a foundation, if the defense objects to the jacket’s admissibility, should the court admit the jacket?

A

The jacket is admissible as relevant evidence linking the defendant to the crime. Generally, all relevant evidence is admissible if offered in an unobjectionable form and manner (i.e., if it does not violate an exclusionary rule, such as hearsay). Clearly, the bloodstained jacket makes it more probably true that the defendant committed the murder than it would have been without the jacket; therefore, the jacket is relevant evidence. Because it does not violate any exclusionary rule, the jacket is admissible

(C) is incorrect because a jacket is not a “statement,” and the hearsay rule excludes out-of-court statements that are offered for their truth. While the wife’s conduct in handing over the jacket arguably may be a statement and perhaps hearsay, the jacket itself is not.

6
Q

What is the full faith and credit doctrine?

A

(C) is incorrect because a jacket is not a “statement,” and the hearsay rule excludes out-of-court statements that are offered for their truth. While the wife’s conduct in handing over the jacket arguably may be a statement and perhaps hearsay, the jacket itself is not.

7
Q

Can you impeach an impeaching witness?

A

Yes, you can impeach any witness

8
Q

Failure to join party….waivable?

A

NO. along with SJ, failure to state a claim those you can bring anytime

9
Q

A corporation, through its president, requested from a financing company a short- term loan of $100,000. On April 1, the corporation’s president and the financing company’s loan officer agreed orally that the financing company would make the loan on the following terms: (1) The loan would be repaid in full on or before the following July 1 and would carry interest at an annual rate of 15 percent (a lawful rate under the applicable usury law); and (2) the corporation’s president would personally guarantee repayment. The loan was approved and made on April 5. The only document evidencing the loan was a memorandum, written and supplied by the financing company and signed by the president of the corporation, that read in its entirety:
“April 5
In consideration of a loan advanced on this date, the corporation hereby promises to pay the financing company, $100,000 on September 1.
The corporation By /s/ the president”
The corporation did not repay the loan on or before July 1, although it had sufficient funds to do so. On July 10, the financing company sued the corporation as principal debtor and the corporation’s president individually as guarantor for $100,000, plus 15 percent interest from April 5.
At the trial, can the financing company prove the president’s oral promise to guarantee the loan?

A

D is correct. There is a written agreement with the president’s signature, but the agreement does not mention the president’s oral promise to personally guarantee the loan. Under the Statute of Frauds, promises to act as a surety or to guarantee the debt of another must meet the writing and signing requirements of the statute to be enforceable. Restatement (Second) of Contracts § 110 (1981). Thus, because the president made the promise orally and it was never written and signed by the president, the president cannot be held liable for the loan amount and the court will not allow the conversation between the loan officer and president to act as proof of the surety agreement.

B is incorrect. According to the Restatement (First) of Contracts § 133 (1932), a person who is neither a promisor nor promisee in a contractual agreement, but stands to benefit from the contract’s performance, is considered an intended beneficiary. Here, the contract was not made to benefit the president. The president’s oral promise to guarantee the loan was actually made to benefit the corporation. Thus, this answer choice mistakes the facts.

10
Q

When and how do you join indespensible parties?

A

n certain situations, a plaintiff must join all interested parties or face dismissal of the lawsuit. The dismissal is usually sought under Rule 12(b) and the issue may be raised any time until judgment. Analysis of a compulsory joinder issue follows a three-step process: (i) should the absentee be joined, (ii) can the absentee be joined, and (iii) if not, should the action proceed in his absence, i.e., is the absentee indispensable.
The absentee should be joined as a party when: (i) complete relief cannot be accorded among the other parties to the lawsuit without the absentee being made a party, or (ii) the absentee has such an interest in the subject matter of the
lawsuit that a decision in his absence will impair or impede his ability to protect the interest or leave any of the other parties subject to a substantial risk of incurring multiple or inconsistent obligations.
Assuming that the absentee should be joined, the next question is whether the court has subject-matter and personal jurisdiction over him. If the court has personal jurisdiction over the absentee, and his joinder will not destroy diversity or venue, he must be joined.
If the absentee cannot be joined, the court must determine whether in equity and good conscience the action should proceed among the parties before it, or should be dismissed, the absentee thus being regarded as indispensable. This decision requires consideration of (i) the extent of prejudice to the absentee or available parties of a judgment, (ii) the extent to which the prejudice can be reduced or avoided by means of protective provisions in the judgment, the shaping of relief, or other measures, (iii) the adequacy of a judgment without the absentee, and (iv) whether the plaintiff will have an adequate remedy if the case is dismissed for nonjoinder.

11
Q

Is a medical examinatino a substantive state issue or procedural?

A

In the famous Erie decision, the Supreme Court held that federal courts sitting in diversity must apply state substantive law on the substantive issues of the case, including state common law. For all procedural issues, federal courts sitting in diversity must apply federal procedural rules, namely the Federal Rules of Civil Procedure (FRCP).
In Sibbach v. Wilson Co., Inc., 312 U.S. 1 (1941), the Supreme Court held that FRCP 35, which outlines the process for ordering a party to submit to a physical or mental evaluation, is a federal procedural law. FRCP 35 is applicable only in certain narrow circumstances, generally when a party’s mental or physical condition is directly at issue in the action. Its use must be supported by good cause, and it cannot be applied to a non-party.

12
Q

The defendant was on trial for murdering his mother, who was found dead in her bathtub. At trial, the prosecutor called the nurse of the defendant’s aunt to testify to what the aunt told the nurse just before the aunt died of cancer. The nurse is prepared to testify that, shortly before she died, the aunt stated, “I know I don’t have much longer to live, so I must tell someone what my nephew said to me yesterday. He told me that he was very angry with his mother and that he wanted to kill her and make it look like an accident!”

Should this testimony be admitted?

A

The testimony is inadmissible. Hearsay is an out-of-court statement offered in evidence to prove the truth of the matter asserted. An out-of-court statement that incorporates other hearsay is “hearsay within hearsay.” Hearsay within hearsay is admissible only if both the outer hearsay statement and the inner hearsay statement fall within an exception to the hearsay rule. Here, the aunt’s statement incorporating the defendant’s statement constitutes hearsay within hearsay. Therefore, both statements must fall within an exception to the hearsay rule to be admissible. The defendant’s statement to his aunt ordinarily would be hearsay because it is being offered to prove the truth of the matter asserted—that he intended to kill his mother. However, statements by a party-opponent (commonly called admissions) are admissible nonhearsay under the Federal Rules. Thus, the defendant’s statement is admissible as a statement by a party-opponent. However, the aunt’s statement made to the nurse regarding the defendant’s admission must also fall within an exception to the hearsay rule. Because no exception applies to that statement, the entire testimony is inadmissible.

(A) is incorrect because, while it correctly states that the defendant’s statement is a statement by an opposing party, it incorrectly deems the statement admissible because it fails to address the hearsay within hearsay issue discussed above.

13
Q

A recent law school graduate was offered a job as an aide by a state legislator. The legislator told the graduate that before she could begin working, she had to take the following loyalty oath: “I swear to uphold our state and federal Constitutions; to show respect for the state and federal flags; and to oppose the overthrow of the government by violent, illegal, or unconstitutional means.” The graduate told the legislator that the oath is unconstitutional and refused to take the oath.

Is the graduate correct?

A

The graduate is correct as to the promise to respect the flag, but incorrect as to the other two promises. The Supreme Court has upheld oaths requiring government employees to oppose the violent overthrow of the government and to support the Constitution; hence, (A) and (C) are incorrect. However, it has held that government employees cannot be required to show respect for the flag, as a person might refuse to salute the flag on religious grounds. Thus, (B) is correct and (D) is incorrect.

14
Q

A homeowner entered into a written agreement with a contractor whereby the contractor agreed to completely remodel the homeowner’s bathroom “to her specifications” at a cost of $10,000. The homeowner’s specifications were highly detailed and required custom-made fixtures that would not be usable in other bathroom remodeling jobs. The contractor ordered the custom-made fixtures and paid $4,000 for them when they were delivered to his place of business. Figuring up the cost of the fixtures and labor, the contractor estimated that he would make a total profit of $2,000 on the job after payment for materials and workers. Before the contractor began work on the project, but after he had paid for the fixtures, the homeowner told the contractor that she had had a change of heart and would probably be selling the house the following year, and so would not need a custom bathroom. The contractor made no attempt to sell the fixtures to another contractor and filed suit against the homeowner for damages.

What is the contractor likely to recover?

A

The contractor can recover $2,000 as lost profits plus the $4,000 in costs he incurred before the homeowner breached the contract. The purpose of a damages remedy based on an affirmance of the contract is to give compensation for the breach; i.e., to put the nonbreaching party where he would have been had the promise been performed. In most cases, the plaintiff’s standard measure of damages will be based solely on an “expectation” measure, i.e., sufficient damages for him to buy a substitute performance. A reliance measure of damages, on the other hand, awards the plaintiff the cost of his performance, i.e., his expenditures in performing his duties under the contract. In certain situations, an award of compensatory damages will contain both an expectation and a reliance component. In a construction contract, if the owner breaches the contract after the builder has already begun his performance, the builder will be entitled to any profit he would have derived from the contract plus any costs he has incurred to date. This formula contains an expectation component (the profit the builder would have made) and a reliance component (the cost incurred prior to the breach). This formula is applicable to the facts in this case. The contractor has begun performance by ordering and purchasing the custom-made fixtures at a cost of $4,000. Because they are usable only for the homeowner’s purposes, their cost, which is treated just like any other expenditure of labor and material in a partially completed construction contract, can be recovered as reliance damages. The other element of his recovery is the $2,000 profit that he would have derived from the contract—his expectation damages. His total recovery will therefore be $6,000. (A) is incorrect because the contractor can do nothing further to mitigate his damages. The nonbreaching party is always under a duty to mitigate damages after learning of the other party’s breach. In construction contracts, the builder’s duty to mitigate generally dictates only that he not continue work after the breach and not incur further expenditures. While the builder would also have a duty to apply any usable materials that he purchased to other jobs or to attempt to resell them to another contractor, the facts specify that the custom-made fixtures here were not usable in other remodeling jobs. Hence, the contractor’s failure to attempt to sell the fixtures did not amount to a failure to mitigate damages.

15
Q

A town adopted an ordinance providing that a person must have been a resident of the town for at least one year to be eligible to vote in school board elections. A resident who moved to the town seven months ago attempted to register to vote in the school board elections scheduled for the next month. However, the town clerk refused to register the resident because he will not have resided in the town for a full year prior to the election. The resident filed a class action suit on behalf of all of the new residents of the town, challenging the validity of the one-year residency requirement.

Which of the following statements is correct?

A

he resident will prevail even if the matter is not decided until after the election, because the suit is not moot and the residency requirement is unconstitutional. The resident’s suit is not moot even if the matter will not be decided until after the election because other members of the class might have a live controversy. Under the case and controversy requirement of the Constitution, there must be a real, live controversy at all stages of the suit. If through the passage of time, the controversy between the parties is resolved, the case is said to be moot. However, there are exceptions to the mootness doctrine. In a class action, it is not necessary that the suit by the named plaintiff be viable at all stages, as long as the claim is viable by some member of the class. Thus, the suit here would not be moot. Moreover, the residency requirement here violates the resident’s fundamental rights to vote and to interstate travel. A restriction on the right to vote is subject to strict scrutiny and is valid only if it is necessary to achieve a compelling state interest (otherwise the restriction violates the Equal Protection Clause by treating new residents differently from old residents). Relatively short residency requirements (e.g., 30 days) have been upheld as being necessary to promote the compelling interest of assuring that only bona fide residents vote. However, the Supreme Court has struck down longer durational requirements for lack of a compelling justification. Thus, the one-year requirement here probably unconstitutionally impinges on the right to vote. The residency requirement also impinges on the fundamental right to travel in the same manner (i.e., it discourages people from migrating by denying them the right to vote without a compelling reason). Thus, the requirement is invalid.

16
Q

Congress passed legislation banning the hunting of snipe birds within the United States. The range of the snipe is quite limited; they are found primarily in only one state, although they migrate annually to several nearby states. Hunters from throughout the United States have traditionally traveled to the snipe’s home state during snipe hunting season, bringing considerable revenue into the state. A state statute allows hunting of snipe during a two-week period in November and charges a $50 license fee for state residents and a $250 fee for hunters from other states. The bag limit is one snipe bird per licensed hunter.

Is the state statute allowing snipe hunting valid?

A

The state statute is invalid because of the Supremacy Clause. Under the Clause, if the federal government adopts legislation that it has the power to adopt, the federal legislation is supreme, and a conflicting state law is rendered invalid. The federal law here, banning the hunting of snipe, is within the federal government’s power under the Commerce Clause, which gives the government power to regulate anything that might affect interstate commerce. Because the birds themselves are found in a few states, they probably cross state lines. Also, hunters come from out of state and generate revenue in the state, so interstate commerce is involved. The state law directly conflicts with the federal law because it allows hunting of snipe. Therefore, the state law will be held invalid under the Supremacy Clause

17
Q

A pedestrian was injured in a car accident involving two cars. The pedestrian filed a negligence action in federal district court against the first driver, seeking $100,000 in damages. The pedestrian is a citizen of State A and the first driver is a citizen of State B. The first driver then filed a third-party claim against the second driver, claiming that the second driver is responsible for half of the harm caused to the pedestrian and seeking to recover half of any liability the first driver is found to have to the pedestrian. The second driver is a citizen of State A.

Does the federal court have subject matter jurisdiction over the third-party claim asserted by the first driver against the second driver?

A

The court has supplemental jurisdiction over the third-party claim. Diversity of citizenship jurisdiction is available when (i) there is complete diversity of citizenship, meaning that each plaintiff must be a citizen of a different state from every defendant; and (ii) the amount in controversy exceeds $75,000. In the instant case, the case is properly in federal court because diversity jurisdiction exists for the underlying claim (i.e., the claim by the pedestrian against the first driver), given that the pedestrian is from State A, the first driver is from State B, and the amount in controversy is $100,000. The third-party indemnity claim, however, cannot invoke diversity jurisdiction, even though the first driver is from State B and the second driver is from State A, because the amount claimed is $50,000.

18
Q

A hockey player who was playing in the final game of the season before a hostile crowd in the opponent’s packed stadium had an opportunity to get his team into the playoffs, but he missed a shot into an open net as the horn sounded, ending the game. As the crowd cheered and jeered, the puck bounced back to him and he shot it in anger toward the stands. A fan who had been looking the other way turned back toward the rink just in time to be struck in the face by the puck. He suffered a broken nose and a severe gash under his eye. After the game, the league commissioner fined the player for violating league rules by intentionally directing the puck out of the playing area.

If the fan sues the player for battery, will the fan likely prevail?

A

The fan will prevail in his battery action because the player had the requisite intent for battery. A prima facie case for battery requires plaintiff to prove (i) an act by defendant that brings about a harmful or offensive contact to the plaintiff’s person, (ii) intent on defendant’s part to bring about harmful or offensive contact, and (iii) causation. The intent element is satisfied as long as the defendant knew with substantial certainty that the harmful or offensive contact would result. Here, the player’s conduct caused a harmful contact to the fan, because the player set into motion the force that caused injury to the fan. His intentionally shooting the puck into the crowded stands is enough to establish that he knew with substantial certainty that the puck would strike a spectator. (Note that even if he only intended to cause apprehension of contact, which is the intent for assault, this intent would suffice for liability for battery under the doctrine of transferred intent.

19
Q

Which of the following suits would not fall within the United States Supreme Court’s original jurisdiction under Article III, Section 2?
A: A suit seeking to assert the interest of state citizens in retaining diplomatic relations with a foreign nation.

B A suit seeking to protect a state’s timber from allegedly illegal cutting by residents of another state.

C A suit seeking to enjoin enforcement of an allegedly unconstitutional executive order that will greatly limit the state’s authority to make policy decisions regarding admission to state universities.

D A suit by the United States Government seeking to enjoin state construction of a bridge over a navigable waterway.

A

The suit to assert state citizens’ rights is not within the Supreme Court’s original jurisdiction. Under Article III, Section 2, the United States Supreme Court has original jurisdiction in all cases affecting ambassadors, other public ministers, and consuls, and in which a state is a party. In (A), the state is not really seeking to advance or protect any interest of its own. Rather, the state is attempting to act in parens patriae (i.e., to act as a representative of its citizens, thereby asserting their interests). Thus, the state is not an actual party in this case in the sense that the Supreme Court has traditionally required to justify exercise of original jurisdiction. (B) would be a proper case for institution under the Supreme Court’s original jurisdiction because it involves an attempt by a state to protect its own economic interest rather than to assert the interests of its citizens in a representative capacity

20
Q

A retailer entered into a written contract with a wholesaler whereby the wholesaler agreed to sell, and the retailer agreed to buy, 100 boxes of sunglasses manufactured by a large corporation located in a neighboring city. The agreed-upon price was $75 per box. Two weeks before the specified delivery date, the wholesaler told the retailer that it would not be able to fill its order, because of unexpected high demand for sunglasses this season. Although the retailer learned that the needed quantity of the same brand of sunglasses could be shipped within two days for $83 per box from a supplier in another area, the retailer instead purchased 100 boxes of the sunglasses locally at a cost of $90 per box. These sunglasses were of a slightly higher quality than the sunglasses that were originally contracted for. A few days before the original delivery date, the wholesaler notified the retailer that it would fill the order, and tendered 100 boxes of the sunglasses on the date of delivery. However, the retailer refused to accept them. At that time, the wholesale market price of the sunglasses had declined to $80 per box.

If the retailer sues the wholesaler for damages based on the wholesaler’s alleged breach, what is the retailer likely to recover?

response

A $1,500, the difference between the cost of cover and the contract price.

Correct
B $800, the difference between the contract price and the nonlocal supplier’s price.

Incorrect
C $500, representing the difference between the contract price and the wholesale market price at the time of performance.

D Nothing, because the retailer obtained cover without waiting a commercially reasonable time for the wholesaler to retract the repudiation.

A

The retailer is entitled to recover $800. The wholesaler’s notice that it would be unable to fill the retailer’s order constituted an anticipatory repudiation, which the retailer was entitled to treat as a total breach. Under the UCC, the buyer’s basic remedy where the seller breaches by refusing to deliver is the difference between the contract price and either the market price or the cost of buying replacement goods (“cover”). If the buyer intends to fix damages based on the latter measure, the buyer must make a reasonable contract for substitute goods in good faith and without unreasonable delay. Here, the retailer chose to make a contract for a higher quality of sunglasses at a higher price, even though the model that he had originally ordered was available from a supplier outside the area. While the retailer need not find the lowest available price in the country or make a contract for substitute goods with an unreliable supplier, he was aware that he could have obtained the sunglasses in plenty of time from the nonlocal supplier. Absent additional facts that would justify the retailer’s decision, he can recover only the difference between the contract cost and a reasonable contract for substitute goods. Hence, (A) is wrong because the retailer’s contract for cover probably would not be deemed to be commercially reasonable. (C) is wrong because the retailer’s remedy based on market price would be determined at the time the retailer learned of the breach, not necessarily the time of performance. In the case of an anticipatory repudiation such as this, the buyer may either treat the anticipatory repudiation as a total breach and pursue his breach of contract remedies, or suspend his performance and await the seller’s performance for a commercially reasonable time. The retailer chose to treat the wholesaler’s notice as a total repudiation and breach of contract. Hence, the market price remedy would be measured at that time because that is when the retailer “learned of the breach,” rather than at the time of performance.

21
Q

Does a covenant to recycle touch and concern the land?

A

NO

22
Q

A defendant visited her doctor to seek treatment for a bullet wound. While he was treating the wound, the doctor asked the defendant how she was shot. The defendant replied that she was struck by a police officer’s bullet while running away from a jewelry store she had robbed, but she implored the doctor not to tell this to anyone. The doctor promised that he would not. Although the defendant was never charged by the police, the owner of the jewelry store brought suit against her seeking the value of the stolen goods. The defendant denied robbing the store. At the trial, the owner calls the doctor to testify to the statement made to him by the defendant. The defense attorney objects on the ground that such testimony is barred by the jurisdiction’s physician-patient privilege.

Should the objection be sustained?

A

NO, not related to treatment

23
Q

A landlord leased office space to a business owner for five years, ending on November 1, reserving a yearly rent of $24,000, payable monthly. On October 1 of the fifth year, the business owner notified the landlord that he was preparing to move, but would greatly appreciate if the landlord could extend the lease for a month or two. On October 10, the landlord wrote to the business owner that she thought they could reach a satisfactory arrangement, but did not hear back from the business owner. The business owner did not vacate the office until November 20. On November 30, the landlord received a check from the business owner in the amount of $1,333 for “November’s rent” and a note that he had vacated the premises.

If the landlord brings an action against the business owner for additional rent, how will the court rule?
A The business owner is bound to a year-to-year tenancy, because he did not vacate the premises until November 20.

B The business owner is bound to a tenancy through December because one month’s advance notice was required to terminate.

C The business owner is not bound, because the $1,333 check discharged him from his obligations.

D The business owner is not bound if the court admits parol evidence of the October 10 letter from the landlord.

A

The court will rule that the business owner is bound to a year-to-year tenancy because he is a hold-over tenant. When a tenant fails to vacate the premises after the termination of his right to possession, the landlord may: (i) treat the hold-over tenant as a trespasser and evict him; or (ii) bind the tenant to a new periodic tenancy. The terms and conditions of the expired tenancy apply to the new tenancy. At least in commercial leases, the new tenancy will be year-to-year if the original lease term was for one year or more. Here, the businessman was a tenant for years because his lease was for a five-year fixed period of time. A tenancy for years ends automatically on its termination date. Therefore, as of November 1, the business owner became a hold-over tenant and the landlord had a right to bind him to a new periodic tenancy. Because the original lease was for more than one year, the business owner may be held to a year-to-year tenancy, at the stipulated rent of $24,000 per year.

24
Q

Example of voting at large:

A city’s water board election laws provide that, although members of the board are elected at large, one member of the board is required to live within each of the five designated water districts within the city. The city’s population was more or less evenly distributed among the districts when this election law was enacted. A resident and registered voter of the city investigated the district residency requirement and discovered that most of the city’s newer residents had moved into the same two water districts, so that the city’s population was no longer evenly distributed among the five water districts. Instead, 80% of the city’s residents lived within its central and eastern water districts, while the other 20% of the city’s residents were scattered among its three other, more rural, districts.

If the resident files suit in federal court challenging the constitutionality of the residency requirement, how will the court most likely rule?

A

The residency requirement is permissible because the water board is elected at large. The Equal Protection Clause prohibits state dilution of the right to vote, so that when a governmental body establishes voting districts for the election of representatives, the number of persons in each district may not vary significantly. This is known as the principle of “one person, one vote.” This principle applies to almost every election where a person is being elected to perform normal governmental functions (e.g., an election for trustees for a junior college district). However, the principle of one person, one vote generally is inapplicable where there is an at-large system of election (except where the system is adopted for discriminatory purposes). Here, the water board members are elected by all of the qualified voters in the city in an at-large system (rather than having the voters of each individual district select one board member apiece), and no discriminatory intent is evident. Thus, the statutory provision requiring board members to reside in each of the five districts does not result in an imbalance or a dilution of the voting rights of the citizens of the city. Consequently, (A) is incorrect, and (D) presents an accurate statement of the constitutionality of the residency requirement. (Note that the answer might be different under federal statute because the city would have to prove a valid, nondiscriminatory purpose.)

25
Q

Treatise ok to impeach and as substantive evidence?

A

YES The Federal Rules recognize an exception to the hearsay rule for learned treatises and admit them as substantive evidence if: (i) the expert is on the stand and it is called to his attention, and (ii) it is established as reliable authority (see above).

26
Q

Wierd question

A defendant is on trial for murder. The only evidence linking the defendant to the crime is some blood found at the scene. The lead detective testifies that an officer took a vial containing a blood sample that had been retrieved by a crime scene technician and drove off with it. The officer is now dead. Next, the prosecution presents as a witness a crime lab chemist. The chemist will testify that he took a vial of blood that contained a label identifying it as having been retrieved from the subject crime scene, and that he performed tests that established a match between that blood and a blood sample taken from the defendant.

Is the testimony of the chemist admissible?

A Yes, because there has been proper authentication.

B Yes, because the chemist qualifies as an expert witness.

C No, because there is insufficient evidence of chain of custody.

D No, because he did not take the original blood sample at the scene of the crime.

A

The testimony is inadmissible because it has not been shown what happened to the blood between the time the officer took it and the time the chemist examined it. Real evidence presents an object in issue directly to the trier of fact. One of the general requirements for admissibility of real evidence is that it be authenticated; i.e., that it be identified as being what its proponent claims it is. If the evidence is of a type that is likely to be confused or can be easily tampered with, the proponent of the object must present evidence of chain of custody. The proponent must show that the object has been held in a substantially unbroken chain of possession. It is not necessary to negate all possibilities of substitution or tampering; rather, what is required is to show adherence to some system of identification and custody. Here, the proponent of the blood sample (the prosecution) has not shown what the officer did with it after leaving the crime scene. There is no showing that the vial was placed directly in a properly secured area so as to diminish the possibility of tampering. In short, it has not been demonstrated that there was adherence to some defined system of identification and custody. In the absence of a substantially unbroken chain of custody, the evidence is inadmissible for lack of proper authentication

27
Q

A tourist and his friend visited an amusement park located in State A. One of the rides malfunctioned, injuring the tourist. The friend, who witnessed the accident, sued the amusement park for negligent infliction of emotional distress in the federal court for State A, properly invoking diversity of citizenship jurisdiction. The friend’s complaint alleges that he and the tourist were lifelong friends and that the friend suffered severe emotional distress from witnessing the tourist’s injuries.

The highest court of State A has held that bystanders may not recover for emotional distress from witnessing another’s injuries unless the bystander and the injured person are related by blood or marriage.

Which of the following motions would best achieve the amusement park’s goal of expeditiously disposing of the action?

A A pre-answer motion to dismiss for failure to state a cause of action.

B A motion for summary judgment.

C A motion for judgment on the pleadings.

D A motion to dismiss for failure to join the tourist as a necessary party.

A

A) The amusement park’s most expeditious course of action would be to file a pre-answer motion to dismiss under Rule 12(b) for failure to state a cause of action. A pre-answer motion to dismiss a complaint for failure to state a claim addresses the adequacy of the pleadings. The allegations of the complaint are deemed true for purposes of the motion. Here, the complaint alleges that the bystander was a lifelong friend of the victim, which is insufficient as a matter of law because State A requires bystanders to be related by blood or marriage to the victim. (B) is incorrect because a party moving for summary judgment submits evidence (typically affidavits and discovery materials) to establish that there are no issues of material fact. Here, the amusement park need not gather or submit evidence because the pleading’s allegations are in themselves insufficient to support the bystander’s claim. (C) is incorrect because a motion for judgment on the pleadings under Rule 12(c) can be made only after the pleadings are closed (i.e., after the defendant files its answer). Thus, the amusement park could more efficiently dispose of the case by making a pre-answer motion to dismiss, rather than filing an answer and then making a motion for judgment on the pleadings.

28
Q

How can one party get the insurance information of the other party?

A

The plaintiff is entitled to obtain information about the defendant’s insurance coverage prior to trial. Without waiting for a discovery request, a party must provide to the other parties copies of insurance agreements under which an insurer might be liable for all or part of any judgment that might be entered.

29
Q

When can you choose to go for the state law method of serving process?

A

Federal Rule 4 provides that summons and complaint may be served on an individual other than an infant or incompetent pursuant to the law of the state in which the district court is located. Notwithstanding, the state provision must be constitutional; i.e., it must be reasonably calculated to give the defendant notice of the action. Thus, (A) is correct

30
Q

A union filed suit against a corporation, known for its antiunion management, asserting that its members were being discharged in retaliation for membership in the union rather than for any failure to perform their jobs properly. Under the pretrial discovery orders, a union employee was allowed to examine all of the records held in the corporation’s files concerning discharge of employees for a seven-year period prior to the instigation of suit by the union. The employee sorted through this large volume of material and discovered that persons who were union activists usually had “lack of corporate spirit” listed as their reason for discharge, while other fired workers tended to have more specific grounds for discharge listed, e.g., persistent lateness. The employee developed a chart showing grounds for dismissal of union members versus nonmembers based on the data in the files. At the trial, the union placed the employee on the stand. She testified in some detail regarding how she had conducted her research. The employee brought out the chart and the union’s lawyer asked that the chart be admitted into evidence. The corporation’s attorney objected.

How should the court rule on the admissibility of the chart?

A Admissible, because copies of the original documents upon which the chart was based were available to the corporation prior to trial.

B Admissible, because the chart is helpful to the trier of fact.

C Inadmissible, because it is hearsay not within any exception

A

The chart is admissible because the original documents are in the corporation’s files. The original document or best evidence rule generally requires the original writing to be produced when the terms of the writing are sought to be proved and are material to the case. [Fed. R. Evid. 1002] However, under Federal Rule 1006, the contents of voluminous writings that are otherwise admissible may be presented in the form of a chart as long as the original documents are available to the other party for examination and copying. Here, the underlying documents belonged to the adverse party, and thus the corporation had unlimited access to them

(C) is incorrect because the chart is admissible provided the underlying documents are admissible. Even if the documents in this case would be hearsay, they would be admissible under the business records exception to the hearsay rule because they are records of events made in the regular course of business

31
Q

A new federal law prohibited the use of various pesticides in areas with a certain population density near navigable waters. A city located in the southeastern United States was plagued by a sharp increase in disease-carrying mosquitoes. The city’s board of health recommended that all residential areas be sprayed with a pesticide proven to be highly effective against mosquitoes. Despite the fact that the federal law would prohibit use of that pesticide in these areas, the city council passed an ordinance adopting the board of health plan, relying on the opinions of several independent experts that the health benefits of reducing the mosquito population outweighed the risks of spraying. An environmentally minded citizen of the city brought an action in federal court challenging the ordinance.

Assuming that the citizen has standing, is the court likely to find the ordinance valid?

A Yes, because pursuant to the police power, cities have a compelling interest in laws designed to protect the health, safety, and welfare of their citizens.

B Yes, because controlling health hazards is an integral governmental function.

C No, because it is superseded by the power of Congress to adopt laws to protect the health, safety, and welfare of citizens.

D No, because it conflicts with a federal law that Congress had the power to make under the Commerce Clause.

A

Congress’s power to regulate commerce has been construed broadly, so that it may regulate any activity, local or interstate, that either in itself or in combination with other activities has a substantial economic effect on interstate commerce. If Congress has determined that the use of chemical pesticides and their runoff into waterways (which are channels of interstate commerce) will have an overall detrimental impact on the environment, this determination will be sufficient in this case to satisfy the standards established by the Supreme Court. Therefore, the law probably is a valid exercise of the commerce power. Any state or local action that conflicts with a valid act of Congress is invalid under the Supremacy Clause. (A) is incorrect because while the police power (the power to adopt regulations for the health, safety, and welfare of citizens) belongs to the states, a police power regulation that conflicts with a federal law is invalid under the Supremacy Clause

32
Q

A cattle-producing state adopted a statute requiring any food service business operating in the state to serve beef raised in the United States. A licensed hot dog vendor who worked at a football field within the state and who had been buying hot dogs made with foreign beef for the past several years estimated that switching to an all-beef hot dog made from United States beef would reduce his profits by 10%. An attorney hired by the vendor to challenge the statute discovered during research into the case that most of the footballs used at the football field at which the vendor worked were made of foreign leather.

Which of the following grounds is the vendor’s best argument against the constitutionality of the state statute?

A

The best argument against the constitutionality of the state statute is that it burdens foreign commerce. For all practical purposes, the power to regulate foreign commerce lies exclusively with Congress. Therefore, a state that adopts legislation requiring private vendors to favor United States products over foreign products, as the state did here, may be acting outside the scope of its powers.

33
Q

WHEN do you remove a case?

A

The court should grant the motion to remand the case because the notice of removal was not timely. Under 28 U.S.C. section 1441(a), a defendant can remove an action that could have originally been brought by the plaintiff in the federal courts. Thus, a case that could have invoked subject matter jurisdiction based on either federal question or diversity of citizenship may be removed from state court to federal. However, removal must be sought within 30 days after receipt by or service on that defendant of the initial pleading or summons, or, if the case is not initially removable, within 30 days of the case becoming removable. Additionally, removal based on diversity jurisdiction cannot be had if more than one year has passed since the suit was filed unless bad faith on the part of the defendant can be shown. Here, the case was immediately removable, and the defendant should have filed a notice of removal 30 days from receipt of service of process, but he did not do so. Accordingly, on a timely motion to remand, therefore, the court should remand the case to state court.

34
Q

A property owner sued a developer in federal court alleging breach of an oral contract. According to the property owner, only one other person of questionable credibility heard the conversation in which the contract was allegedly made. That person was listed in the pretrial conference order. The next day, five days before the scheduled trial, the property owner’s attorney discovered that a disinterested person also heard the conversation, and he wanted to call this witness at trial as well. He immediately notified the developer of the witness and his change in plans.

May the property owner call the additional witness?

A

YES if the COurt modifies the pre trial order.
The property owner may call the additional witness only if the court modifies the pretrial order, since a pretrial order controls the subsequent course of an action unless modified. The order will be modified “only to prevent manifest injustice.” [Fed. R. Civ. P. 16(e)]

35
Q

A man and a woman were in a two-car traffic accident. Immediately after the accident, the man was treated by a physician at a nearby hospital’s emergency room. The man later filed a negligence action against the woman in federal district court, seeking compensatory damages. The woman now seeks discovery regarding the emergency room physician’s observations, opinions, and treatment of the man.

Is the woman entitled to discovery regarding that information?

A

The woman is entitled to discovery regarding the physician’s observations, opinions, and treatment of the man because the physician developed opinions about the man’s injuries for purposes other than litigation or trial.

(C) is incorrect because it is not applicable to the situation here, where the physician was not retained in anticipation of litigation or trial.

36
Q

When can you get default from a default judgement?

A

An entry of default may be set aside for “good cause shown.” Although not specifically required by the Federal Rules, a majority of courts also will require some showing of a meritorious defense.

37
Q

Is this OK:

“INSTRUCTION #6: In order to mitigate an intentional killing to voluntary manslaughter, the burden of proof is on the defendant to establish that adequate provocation existed.”

A

NO, prosecution must prove every element of the crime.

The court should reverse the defendant’s conviction because Instruction #6 requires the defendant to disprove one of the elements of murder. Due process requires in criminal cases that the state prove guilt beyond a reasonable doubt. The prosecution has the burden of proving all of the elements of the crime charged. Thus, if malice aforethought is an element of murder and voluntary manslaughter is distinguished from murder by the existence of adequate provocation, the defendant cannot be required to prove that he committed the homicide in the heat of passion (i.e., with adequate provocation). Such a requirement would impose on the defendant the burden of disproving the element of malice aforethought, because “heat of passion” negates malice. Although the defendant can be given the burden of going forward with some evidence on the provocation issue, once he has done so, the prosecution bears the burden of proving that the killing was not done in the heat of passion. In the case at issue, Instruction #6 requires a defendant to prove that he committed the intentional killing under adequate provocation. At common law, and consequently in the state, malice aforethought is an element of murder. Therefore, this instruction in effect requires the defendant to disprove the element of malice aforethought, thereby relieving the state of its burden of proving all elements of the crime. As discussed above, such an instruction cannot pass constitutional muster. On the other hand, for an affirmative defense such as insanity, it is permissible to impose the burden of proof on the defendant. Thus, Instruction #8 does not affect the state’s obligation to prove all elements of the crime, and is permissible under the general principles mentioned above

38
Q

Can lay person testify as to the authenticity of a signature when they have seen the dude sign only once in another unrelated occasion?

A

YES The court should find this testimony admissible. Any person can testify to the authenticity of another’s signature as long as that witness has seen the person’s signature and can express an opinion regarding its authenticity. The only restriction is that a nonexpert cannot become familiar with the handwriting merely for the purpose of testifying, and this was not the case here. There is no requirement that the witness have seen the signature recently;

39
Q

Can Pres give blanket pardons? (all your crimes in past 20 years are ok)

A

YES.

40
Q

Tell me about accomplice liabiltiy

A

Principals are those who, with the requisite mental state, actually engage in the act or omission that causes the criminal result. An accomplice is one who, with the intent that the crime be committed, aids, counsels, or encourages the principal before or during the commission of the offense. Under modern statutes, accomplices are generally treated as principals. A third category is “an accessory after the fact.” An accessory after the fact is one who receives, relieves, comforts, or assists another knowing that he has committed a felony, in order to help the felon escape arrest, trial, or conviction. Unlike an accomplice, an accessory after the fact has committed a separate crime with a punishment unrelated to the felony committed. In the instant case, it is clear that the defendant aided his friend in avoiding capture, that he provided no aid to the substantive offense, and that he did not intend the substantive offense to occur.

41
Q

A proud grandfather who planned to take pictures of his grandson’s graduation purchased a camera from a camera store. He used the camera on several occasions over the next few weeks without incident, but when he used it on the day before his grandson’s graduation, it caught fire and exploded, burning him and destroying an expensive coat he was wearing. Although the grandfather was in a great deal of pain because of his injuries, he insisted on attending his grandson’s graduation. However, because he no longer had a workable camera, the grandfather hired a professional photographer to take pictures of the special day.

In a breach of warranty action, which of the following represents the most that the grandfather may recover?

A

When a buyer accepts goods that turn out to be defective, he may recover as damages any “loss resulting in the normal course of events from the breach,” which includes the difference between the value of the goods accepted and the value they would have had if they had been as warranted, plus incidental and consequential damages. Incidental damages resulting from the seller’s breach include expenses reasonably incurred in inspection, receipt, and transportation, care, and custody of goods rightfully rejected. In this case, the grandfather incurred no incidental damages. Consequential damages resulting from the seller’s breach include any loss resulting from general or particular requirements and needs of which the seller at the time of contracting had reason to know and which could not reasonably be prevented by cover or otherwise, and injury to person or property proximately resulting from any breach of warranty. Here, the grandfather is entitled to breach of warranty damages for the loss of the camera—the difference between the value of the camera accepted and its value if it had been as warranted—plus damages for injury to his person (e.g., medical costs for treating the grandfather’s burns) and property (i.e., the cost to replace his coat) because they were proximately caused from the breach of warranty. Thus, (C) is correct,

42
Q

A mother received a telephone call asking her to meet with the vice principal of her son’s school because he was caught cheating. As she was waiting in the hallway to meet with the vice principal, the school’s janitor, a big man who was an amateur body builder, introduced himself as the vice principal and asked the mother to step into his office, closing the door behind him. For several minutes, the janitor asked the mother a series of flirtatious personal questions, causing the mother to grow very uncomfortable. However, she did not leave the office out of concern for her son’s academic standing. The actual vice principal eventually returned to his office and ended the questioning.

If the mother sues the janitor for false imprisonment, is she likely to prevail?

A

he mother likely will not prevail in a false imprisonment action against the janitor because the mother was in the room willingly and no facts show that she was there against her will. For false imprisonment, a plaintiff must prove that the defendant intentionally confined or restrained plaintiff to a bounded area, and here there is no evidence of that. (A) is wrong because, although a plaintiff need not risk harm by trying to escape, the mother was not trying to escape. (B) is incorrect because, whatever the reason for her staying in the room, there are no facts showing that the janitor intended to falsely imprison her.

43
Q

After picking up a load of hazardous chemical waste, a truck driver for a waste management company set out on the road to his next stop. However, he had failed to secure the latch on the back panel of the truck. Consequently, the panel opened while the truck was on the road, and a metal canister full of chemical waste fell onto the road. A car struck the canister, causing the car to veer off the road and injure the driver. The driver filed suit against the company for his injuries.

The jurisdiction in which the above events took place has adopted a rule of partial comparative negligence. At trial, the driver of the car admitted that he had momentarily taken his eyes off the road to look at his speedometer. When he had looked up again, the canister was there and he could not stop in time. The jury found that the company, through its truck driver, had acted willfully and wantonly and was 90% at fault, while the driver of the car was 10% at fault. The driver filed a motion for judgment notwithstanding the verdict, seeking recovery for 100% of his damages.

If the judge grants the motion, what is the most likely reason?

A

If the driver was effectively required by statute to take an occasional quick look at his speedometer to make sure that he was complying with appropriate speed limits, then his momentary glance at the speedometer in the instant case would, as a matter of law, not constitute negligent conduct. Because this is a matter of law, the judge would be authorized to correct this aspect of the jury’s verdict. If the driver is thus found to be not negligent in this matter, his recovery will not be reduced

(D) is incorrect because, although the transportation of chemical waste would probably be considered an abnormally dangerous activity, liability for conducting an abnormally dangerous activity attaches only if the harm results from the kind of danger to be anticipated from such activity; i.e., the injury must flow from the normally dangerous propensity of the activity. The canister falling from the truck is not the “normally dangerous propensity” of transporting chemical waste.

44
Q

Is there res judicata here?

A landlord brought suit against a tenant in federal court for overdue rent payments on a commercial lease. The landlord sought to recover on the six rent installments that were past due and unpaid at the time of the suit. The landlord won the case, and judgment was entered in her favor. The lease has an acceleration clause that states that all future rent payments become due if the tenant falls behind three months or more. The landlord now files suit against the tenant for the remaining rent payments. The tenant moves to dismiss, asserting that the landlord’s claim is barred by claim preclusion (res judicata) principles.

Should the tenant’s motion to dismiss be granted?

A

YES, same transaction or occurrence.

45
Q

When can a third party intervene?

A

Under Rule 24, a nonparty may intervene in an action as a matter of right in two situations. First, a nonparty may intervene when it has an unconditional right to do so by a federal statute. Second, a nonparty may intervene if (i) it has an interest in the property or transaction the is the subject matter of the action; (ii) the disposition or resolution of the action may as a practical matter impair the nonparty’s ability to protect its interest; and (iii) the nonparty’s interest is not adequately protected by an existing party in the action.

46
Q

A developer owned a 240-acre parcel of land zoned for commercial and residential use. He prepared and recorded, after obtaining approval from all appropriate agencies, a subdivision plan that included a commercial center and a number of lots for single- and multi-family residences. The list of covenants, conditions, and restrictions recorded with the plan included provisions that required every building constructed in the subdivision to be of “simulated adobe style” architecture approved in advance by an association. A year later, the developer sold many of the lots in the commercial center, including several to a real estate firm. Each deed prepared by the developer contained a reference to the design restriction in the recorded plan. The developer also sold almost all of the residential lots, the deeds of which contained the same reference to the restriction. The following year, the real estate firm sold one of its lots to a burger franchise. The deed contained no reference to the design restriction. The franchise’s prefabricated restaurant, complete with a giant burger logo mounted on the roof, was constructed over the weekend.

A merchant, an original purchaser of one of the commercial lots, owned the lot next to the burger franchise. She did not learn of construction of the restaurant until she came in to work on Monday, and saw the giant burger logo. The merchant brings an action seeking a mandatory injunction compelling the burger franchise to demolish the restaurant. At trial, the merchant proves that the burger franchise did not seek or obtain approval of the association for its building.

Should the court issue the injunction?

A No, because destruction of the restaurant would be a tremendous waste of resources.

B No, because the burger franchise’s deed contained no restriction on the type of building that could be constructed on the lot.

C Yes, because the restrictive covenant runs with the land.

D Yes, unless the burger franchise can establish to the court’s satisfaction that its restaurant design has at least as much aesthetic merit as any “simulated adobe style” design.

A

[short answer: requiring certain aestetic runs with the land]

The court should issue the injunction because the covenant runs with the land. A covenant will be enforceable as an equitable servitude—allowing a covenantee, covenantor, or successor to enforce the covenant in equity by way of injunction—when there is (i) a covenant in a writing satisfying the Statute of Frauds, that (ii) touches and concerns the land (i.e., the effect of the covenant makes the land more useful or valuable to the benefited party) and that (iii) indicates an intention that the servitude exists, and (iv) notice is given to future owners of the burdened land. Here, the covenant was in writing in the subdivision plan and presumably it satisfied the Statute of Frauds. It touches and concerns the land—benefiting all of the lots and burdening all of the lots. The intention to create the servitude is established by the writing and can also be implied from the common scheme for development. There was sufficient record notice of the covenant because the plan was recorded and was noted in all of the original deeds prepared by the developer, including the one in the burger franchise’s chain of title.

47
Q

A defendant is on trial for violating a statute forbidding possession of a concealed weapon within 100 yards of a government building. The prosecution presents evidence that the defendant was arrested on a street corner with a handgun in his pocket. The building housing the local city hall occupies the entire block on the north and east sides of the two streets where the defendant was apprehended.

Which of the following statements is most accurate regarding judicial notice of the location of the city hall?

A

The judge may take judicial notice of this fact because it is a matter of common knowledge in the community, but the jury is not required to accept the fact as conclusive in a criminal case. Judicial notice may be taken of facts that are not subject to reasonable dispute because they are generally known within the territorial jurisdiction of the trial court. [Fed. R. Evid. 201(b)] The facts need not be known everywhere as long as they are known in the community where the court is sitting. The location of the city hall is such a fact. As choice (A) also states, in a criminal case the jury should be instructed that it may, but is not required to, accept as conclusive any fact that is judicially noticed. [Fed. R. Evid. 201(f)] (B) is incorrect. While facts that are not generally known and accepted may be a subject of judicial notice if they are easily verified by resorting to easily accessible, well-established sources (i.e., facts capable of certain verification), facts that are matters of common knowledge in the community, such as the location of the city hall, may be judicially noticed without resort to reference materials.

48
Q

Establishment clause and books

A

The Supreme Court has held that a state lending textbooks on secular subjects to all students, including those at religious schools, does not violate the Establishment Clause.

49
Q

Dude charged with murder. Then indicted with conspiracy. OK?

A

YES. The defendant’s motion should be denied because a prosecution for conspiracy is distinct from a prosecution for any substantive offense involving the same conduct as the conspiracy. The Fifth Amendment provides that no person shall be twice put in jeopardy for the same offense. The general rule is that two crimes do not constitute the same offense if each crime requires proof of an additional element that the other crime does not require, even though some of the same facts may be necessary to prove both crimes. [Blockburger v. United States (1932)] Furthermore, a prosecution for conspiracy is not barred merely because some of the alleged overt acts of that conspiracy have already been prosecuted. [United States v. Felix (1992)] Here, both the conspiracy charge and the possession charge require proof of an element that the other charge does not; hence, there is no double jeopardy problem with the indictment.

50
Q

A driver and his passenger were involved in an automobile accident when the driver ran a red light and crashed into another car. Due to a manufacturing defect in the automobile’s airbag system, the passenger side airbag did not deploy. The passenger was killed on impact. The passenger’s estate brought suit against the driver and the airbag’s manufacturer. At trial it is established that the driver was negligent in running the red light.

What effect would such proof have on the claim of the passenger’s estate against the airbag manufacturer?

A

The driver’s negligence would bar recovery if it was the sole legal cause of the passenger’s death. Regardless of the theory that the plaintiff is using in a products liability action, actual and proximate cause must be established. If the driver’s negligence is the sole legal or proximate cause of the passenger’s death, it would preclude the estate’s suit against the airbag manufacturer because the defect was not a legal cause of the passenger’s death.

(D) is incorrect. In most pure comparative negligence jurisdictions, the same comparative fault rules will apply whether the action against the manufacturer is based on negligence or strict liability. As discussed above, whether the estate can recover, and the extent of its recovery, depends on causation issues and the driver’s status as a beneficiary of the estate.

51
Q

A fashion student at a prestigious fashion design school bought a new sewing machine for $1,000 so that she would be more than adequately equipped for her design assignments. One day, her roommate loaned the sewing machine to their neighbor, as she had done on several prior occasions. Unfortunately, the neighbor caused extensive damage to the machine. The cost to repair the sewing machine was $400.

If the fashion student sues her roommate for the damage the neighbor caused to the sewing machine, what will be the result?

A

it’s conversion and you don’t get 1000 but the “fair market value of the good” [yes, the answer did not say at the time….]

52
Q

In an attempt to induce her niece to improve her grades, an aunt told her niece that she would give her $50 for every “A” she earned in law school. Inspired by the opportunity to earn extra spending money, the niece studied like never before and ended up earning three “A’s” the following semester. Upon contacting her aunt with the news, the niece learned that her aunt was “only kidding” about paying her for every “A” earned in law school.

If the aunt’s offer to her niece was held to be valid, it would be an offer for what type of contract?

A

The aunt’s offer to her niece was for a unilateral contract. In a unilateral contract, acceptance of an offer is possible only by performing a stipulated act, whereas in a bilateral contract, acceptance of an offer is accomplished by promising to do the stipulated act. The aunt’s offer could be accepted by the niece only by completion of performance (i.e., obtaining an “A” in the class), and when the niece received her three “A’s,” a unilateral contract was formed

53
Q

Remove- if you in northern CA state can you remove to southern ca?

A

NO has to be same district and division

54
Q

A man was tried in state court for possession of heroin. The prosecution offered in evidence five rolled-up toy balloons containing heroin, which police officers had found on a table in the man’s apartment. At a hearing on the defense’s motion to suppress, testimony was presented that established that the police had put the apartment under surveillance and had watched a police informant go to the door of the apartment, hand four balloons of heroin to the man, and leave. The police had then knocked on the apartment door, identified themselves as police officers, and demanded entrance. Having heard nothing for 30 seconds, the police had then broken down the door and entered the apartment, discovering the heroin. The police had intended to arrest the man for the purchase of heroin, a felony. When they had gotten inside the apartment, they discovered that the man had left by a back exit. He was later arrested at the nearby newsstand.

The trial court denied the motion to suppress, and the case is on appeal following the man’s conviction for possession of heroin. How should the appellate court rule?

A

The appellate court should reverse the conviction on Fourth Amendment grounds. In Payton v. New York (1980), the United States Supreme Court held that, absent an emergency, a forcible, warrantless entry into a residence for the purpose of making a felony arrest is an unconstitutional violation of the Fourth Amendment as made applicable to the states by the Fourteenth Amendment. No exigent circumstances justified the warrantless arrest or the warrantless entry into the home. If the police had reason to believe the man was destroying evidence, they could have entered the home without a warrant to prevent the destruction under the exigent circumstances doctrine. But nothing in the facts gave the officers reason to believe that evidence was being destroyed. They knocked on the door, identified themselves, and demanded entrance. They heard no response or sounds of drugs being destroyed. Thus, there were no exigent circumstances. Evidence that is the fruit of an unlawful arrest may not be used against the defendant at trial because of the exclusionary rule.

55
Q

I giv you a quitclaim deed. After 5 years I tell you to give it back to me and you do and I destroy the deed. OK?

A

NO can’t take back delivery (you would have to go to a court to nullify it)

56
Q

Lay opinion

A

The orderly’s testimony should be admitted because it is proper opinion testimony by a lay witness. Where an event is likely to be perceived as a whole impression, rather than as more specific components, opinions by lay witnesses are generally admitted. Lay opinion testimony is admissible when: (i) it is rationally based on the perception of the witness; (ii) it is helpful to a clear understanding of his testimony or to the determination of a fact in issue; and (iii) it is not based on scientific, technical, or other specialized knowledge. [Fed. R. Evid. 701] One matter about which a lay witness may testify is the general appearance or condition of a person

57
Q

Strict liability - causation issue:

A child was severely injured at an amusement park when she was ejected from a ride that went slightly off its track. The ride malfunctioned as a result of a manufacturer’s defect, but had the child been properly secured in the ride’s seatbelt by one of the ride operators, she would not have been injured. The child was unable to identify which ride operator improperly buckled her in.

In the child’s suit against the amusement park, who will win?

A

Because the ride operator was negligent in improperly securing the child, the amusement park is vicariously liable under the doctrine of respondeat superior. This doctrine imposes liability on an employer for the tortious conduct of its employee occurring within the scope of the employment relationship. Here, securing the children in the ride was one of the ride operator’s tasks. Since this task was performed negligently and this negligence was one of the causes of the child’s injuries, the amusement park will be liable.

58
Q

little revision on invasion of privacy

A

To establish a prima facie case for invasion of privacy based on publication by defendant of facts placing plaintiff in a false light, the following elements must be proved: (i) publication of facts about plaintiff by defendant placing plaintiff in a false light in the public eye; and (ii) the “false light” must be something that would be highly offensive to a reasonable person under the circumstances. Here, the photo created the false impression that the minister was exiting an adult bookstore. Publication of the photo conveying this false impression of the minister’s conduct would be highly offensive to a reasonable person under the circumstances.

59
Q

A yoga instructor entered into a valid written contract with a builder to construct a large yoga studio on some land she owned outside of town. She agreed to pay the builder $150,000 upon completion of the job. As work progressed, and due to substantial increased building costs, the yoga instructor and the builder orally agreed that the builder may omit installation of the koi pond planned for the atrium (saving the builder $1,000), and that the contract price would be reduced to $149,500. The builder completed the job (minus the koi pond) in reliance thereon.

What would most courts likely hold this subsequent oral agreement to be?

A

The agreement is enforceable because both the builder and the yoga instructor gave new consideration to support the modification. If parties agree to modify their contract, consideration is usually found to exist where the obligations of both parties are varied. It is usually immaterial how slight the change is, because courts are anxious to avoid the preexisting duty rule. Here, the obligations of both the builder and the yoga instructor are varied—he will not install the koi pond and she will pay a construction price reduced by $500. Consideration is therefore found in the promise of both parties to forgo their rights under the original contract—the builder’s right to full contract price and the yoga instructor’s right to the koi pond. (

60
Q

A grandfather told his granddaughter that she could have his house because he was moving to a retirement home, and entered into a valid contract to convey it to her. He promised her that he would have another wing added to the house in the back before turning it over to her, and entered into a written contract with a builder to construct the addition for his granddaughter. Before the grandfather had entered into the contract with the builder, the granddaughter had paid $5,000 for a 60-day option to purchase another house because she was not sure she would like the addition. However, when her grandfather showed her the plans for his house prepared by the builder, she liked it very much and decided to let her option to purchase the other house lapse. Shortly thereafter, the local zoning authority increased the minimum lot line setbacks, making it impracticable to put the addition on the back of the house. The builder offered to put an addition above the existing floor rather than in the back, and the grandfather agreed. After the granddaughter’s option had lapsed, she discovered that the addition was now going up rather than in the back. She angrily demanded that the builder either build the addition according to the original specifications that she approved or pay her damages. The builder refused and the granddaughter filed suit.

Who is more likely to prevail?

A

The builder will prevail because he may raise all defenses that he had against the grandfather against the granddaughter. The granddaughter is an intended third-party beneficiary of the contract between the grandfather and the builder. Generally, a third-party beneficiary has rights under the contract as soon as she does something to vest her rights (manifests assent to the promise, brings suit to enforce the promise, or materially changes position by justifiably relying on the promise). Here, the granddaughter materially changed her position by justifiably allowing her option on the other house to lapse. Generally, once the third-party beneficiary’s rights have vested, the original contracting parties may not modify the contract without the assent of the third-party beneficiary. However, the third-party beneficiary is subject to any defenses that the promisor could have used against the original promisee, and here the builder could have used the defense of impracticability against the promisee. Therefore, he could use that defense against the granddaughter to avoid having to pay damages for not building the house as he originally agreed.

61
Q

The plaintiff was severely injured when a tire on her car blew out and caused her to lose control of the car. She filed an action in federal district court against the manufacturer of the tire, alleging that the tire was defective. The plaintiff intends to introduce remnants of the tire into evidence at trial as part of her proof that the tire was defective.

In what way are the remnants subject to discovery?

A

Without waiting for a discovery request, a party must provide to the other parties copies or descriptions of tangible things that are in the disclosing party’s possession or control and that the disclosing party may use to support its claims or defenses as an initial disclosure under Rule 26(a)(1). (Separately, the tire remnants would have to be disclosed under Rule 26(a)(3) as a pretrial disclosure of an item that is expected to be offered into evidence.)

62
Q

Can you voluntarily dismiss after D has answered with a motion or answer?

A

Yes but…
If the defendant has answered or filed a motion for summary judgment or there was a previous dismissal, the plaintiff must file a motion for voluntary dismissal by leave of the court, and the court has the discretion to grant dismissal on such terms and conditions as the court deems proper. Here, because the distributor answered the complaint, this motion for voluntary dismissal with leave of the court was a proper vehicle to attempt to achieve the storeowner’s goal; however, due to the excessive time (14 months) and expense (over $200,000), it is unlikely that the court would deem these conditions proper to grant such a dismissal.

63
Q

The question was about a prior creditor who wanted to be protected by a notice statute

A

The victim will not likely prevail against the bank because a majority of courts hold that the judgment lienor is not protected by the recording statute. If the statute here, which is a notice statute, were applicable to protect the victim, he would have priority over the bank because his judgment lien was recorded before the buyer’s deed was recorded. Under this view, the bank’s mortgage would have been considered “wild” and would be deemed unrecorded because the preceding conveyance, the buyer’s deed, was actually unrecorded. A searcher in the public records would therefore have been unable to find the mortgage. Hence, if the statute were applicable to protect the victim, he would have priority over the bank. However, most courts reason that either (i) a judgment creditor is not a bona fide purchaser because he did not pay contemporaneous value for the judgment, or (ii) the judgment attaches only to property “owned” by the debtor, and not to property previously conveyed away, even if that conveyance was not recorded. Under the statute in the present question, a judgment does not attach until it is recorded. Here, the victim’s judgment did not attach to the land until after the bank obtained a mortgage on it, and the recording statute does not change that result. The failure of the buyer to record, and the resultant treatment of the bank as unrecorded, is irrelevant. Thus, the bank’s mortgage is superior to the victim’s lien.

64
Q

Where does the public policy exception of settlements apply?

A

(C) is wrong because there is no attempt here to offer a statement made during the plea bargaining process. Under the Federal Rules, withdrawn guilty pleas, pleas of nolo contendere, offers to plead guilty, and evidence of statements made in negotiating such pleas are not admissible in any proceeding. [Fed. R. Evid. 410] However, there is no prohibition against admitting the guilty plea itself. This question asks whether the guilty plea is admissible, not whether statements made in negotiation thereof are admissible. Thus, the policy favoring plea bargains is irrelevant.

65
Q

Investigation on President’s possible embezzlement. Can President say “executive privilege on papers”

A

There is privilege but documents would come in since we must account for government interests as well.

66
Q

Dormant Commerce Clause

A

is incorrect because states may regulate local aspects of interstate commerce in the absence of federal regulation as long as the regulation is nondiscriminatory and does not unduly burden interstate commerce, which is a case-by-case balancing test. Here, the legislation appears to be nondiscriminatory and there are insufficient facts to establish that it would constitute an undue burden; hence, (B) presents a stronger argument than (D).

67
Q

A comprehensive federal health-care reform statute created a Federal Health Policy Board, which was directed to monitor the fees charged for various medical procedures covered by insurance. The board also had the power to subpoena records to determine whether fee increases were a true reflection of cost increases. Nothing in the statute provided for caps on fee increases.

Because of the continuing escalation of health-care costs while the statute was being debated, several states had passed health-care legislation on their own. One state passed legislation that prohibited most fee increases of 10% or more per year for specified health-care services covered by insurance, and created a health-care review board to regulate these costs and impose monetary penalties on health-care providers or insurers that tried to circumvent the cap.

Which of the following would be the best basis for finding the state provision unconstitutional

A

The fact that the federal board was similar to the state board but was not given the power to restrict fee increases and impose sanctions in an otherwise comprehensive bill suggests that such provisions in the state law violate the Supremacy Clause. A state law may fail under the Supremacy Clause even if it does not directly conflict with a federal statute or regulation if it interferes with the achievement of a federal objective or the federal regulations occupy the entire field. Where the federal laws are comprehensive or a federal agency is created to oversee the field, preemption will often be found. The fact that the health-care legislation was comprehensive but the federal board was not given regulatory or enforcement power suggests that Congress did not want specific restrictions in these areas and may have wanted free-market principles to determine fee increases at the outset. The state board’s power to impose these restrictions may violate the Supremacy Clause under these circumstances. (A) is incorrect because the fact that the federal legislation was passed later does not automatically mean that the state legislation has been superseded. In areas of concurrent legislative power, a state regulation will be upheld if it does not conflict with and is not preempted by federal legislation

68
Q

A thief sold some stolen goods to a dealer. Several weeks later, the police raided the dealer’s store and arrested him. In this raid, the police seized the goods the thief sold to the dealer and a record book in which the dealer had recorded this transaction. However, at the dealer’s subsequent trial for receiving stolen goods, the charges against him were dismissed when the court ruled that the search warrant had been improperly issued.

The police were able to trace the stolen goods to the thief because of fingerprint identification and the information contained in the dealer’s record book.

At his trial, the thief made a motion to suppress the stolen goods and record book.

What should the judge do?

A

The court should deny the motion because the thief had no standing to object to the search. A person challenging the admissibility of seized evidence must have standing to do so. As a general rule, standing requires a person to have a reasonable expectation of privacy in the place being searched or the item being seized. One may not challenge a search or seizure by claiming that another person’s constitutional rights have been violated. Here, the thief had no ownership interest in the dealer’s store. He had no reasonable expectation of privacy with respect to it; i.e., he was not present when the search was made, and he had no ownership interest in the stolen goods. Thus, he lacks the standing to object to their illegal seizure.

69
Q

A company that was the leading supplier of home water filtration systems had a network of sales promoters who were under contract for two- or three-year terms and were compensated solely by commissions earned from sales and by occasional bonuses. Veteran promoters also earned commissions by recruiting other promoters for the company. One of the company’s veteran promoters was contacted by a former top sales representative for another manufacturer who was looking for similar sales opportunities in the region. The sales rep knew that the promoter might be able to get her a position with his company, which was looking for additional promoters. At the time he met with the sales rep, the promoter’s contract with the company had one more month to run. When the promoter’s contract with the company expired, he announced that he was forming his own business to market a different line of water filtration systems manufactured by a competitor of the company, and that the sales rep would be in charge of his promotional network.

The company brought an action against the promoter for interference with business relations for hiring the sales rep. At a preliminary hearing, the parties stipulated to the above facts and that the promoter was an independent contractor rather than an employee of the company. The promoter then filed a motion for a summary judgment in his favor.

Should the court grant the promoter’s motion?

A Yes, because the sales rep had no business relationship with the company at the time the promoter’s alleged interference occurred.

B Yes, because the promoter was an independent contractor rather than an employee of the company.

C No, because the jury could find that the means the promoter used to obtain the sales rep were not privileged.

D No, because the jury could find that the promoter breached his contract with the company by meeting with the sales rep.

A

NOT A:
A) is incorrect because even though the company did not have an existing contractual relationship with the sales rep, it could very well show that it had a reasonable expectation of signing a contract with the sales rep that the promoter knew of and intentionally interfered with. (Note that courts do not permit recovery for negligent interference with business relations.) Whether the company could prove its expectancy to a sufficient degree to establish actual damages would be a question for the trier of fact; hence, summary judgment would not be appropriate on this basis.

C is CORRECT:
The court should not grant the promoter’s motion because the jury could find that the promoter used improper means, while working for the company, to divert the sales rep for his own purposes. To establish a prima facie case for interference with business relations, the following elements must be proved: (i) existence of a valid contractual relationship between plaintiff and a third party or a valid business expectancy of plaintiff; (ii) defendant’s knowledge of the relationship or expectancy; (iii) intentional interference by defendant that induces a breach or termination of the relationship or expectancy; and (iv) damage to plaintiff. Thus, a plaintiff has a cause of action for interference with probable future business relationships for which the plaintiff has a reasonable expectation of financial benefit. On the other hand, an interferer’s conduct may be privileged where it is a proper attempt to obtain business for the interferer, particularly if the interference is only with a prospective business relationship rather than with an existing contract. What is proper depends on various factors, including the means of persuasion used. Here, the promoter’s conduct would not be privileged if the jury were to find that he improperly used his position with the company to develop a relationship with the sales rep.

70
Q

Does a clerical error automatically amount to a mutual mistake?

A

NO, dah is incorrect because the mistake regarding the delivery date is a mere clerical error, rather than a mistake of fact going to a point that is material to the transaction. As such, this is not the type of mistake that would relieve either or both of the parties of their obligations under the contract.

71
Q

A homeowner hired a contractor to make some improvements on his house. They entered into a written contract providing that the contractor would do the improvements for $5,000. Shortly after the contract was signed, the contractor told the homeowner to give the money to his (the contractor’s) daughter when the job was finished, adding, “She is getting married soon and I want her to have a nice wedding present from me.” The daughter was aware that her father made this statement to the homeowner. She married, but soon thereafter the contractor told the homeowner to pay him the $5,000, and not the daughter, because his son-in-law had a gambling problem and would probably use the money to bet at the racetrack.

What is the best argument in favor of the daughter’s being able to enforce a contract for $5,000 in her favor?

A Statute of Frauds.

B Parol evidence rule.

C The daughter was an intended third-party beneficiary.

D The daughter married in reliance on the promise.

A

C does not help her because she was not an intended third-party beneficiary. If a contract between two parties contemplates performance to a third party, that third party may have rights to enforce the contract. To do so, the third party must be an intended beneficiary at the time the contract was made (e.g., designated in the contract). An assignment, on the other hand, is a contract that does not contemplate performance to a third party when the contract is made. Rather, later one of the parties transfers his rights to another. Here, the contractor and the homeowner signed their contract and later the contractor assigned his rights to his daughter. Thus, the daughter was not an intended third-party beneficiary who could enforce the agreement, but merely an assignee who gave no consideration for the assignment. As such, the contractor was free to revoke the assignment, and his daughter cannot recover the $5,000.

[it was the reliance thing]

72
Q

Can you depose a corporation?

A

When giving notice of deposition to an organization, a party may name the organization and state with reasonable particularity the matters to be covered. The organization then should designate individuals to testify on its behalf

73
Q

A landowner and his friend owned a tract of land as joint tenants with right of survivorship. The landowner executed a deed conveying his interest in the land to his grandson. The landowner gave the deed to his attorney with instructions to deliver it to the grandson upon the landowner’s death. The grandson first learned of the deed at the landowner’s funeral the following year. The next day, the grandson recorded the deed.

Who owns the land?

A The friend and the grandson, as joint tenants.

B The friend and the grandson, as tenants in common.

C The friend.

D The grandson.

A

The friend owns the land. A joint tenancy is a concurrent estate in land in which each co-tenant has an undivided right in the property and a right of survivorship—when one joint tenant dies, the property is freed of his concurrent interest and the survivor retains an undivided right in the property that is no longer subject to the interest of the deceased co-tenant. An inter vivos conveyance by one joint tenant, even a “secret” deed that is to take effect only upon the grantor’s death, severs a joint tenancy. However, although acceptance (presumed or otherwise) usually “relates back” to the date of delivery of the deed in escrow, many courts refuse to relate back an acceptance where it would defeat the rights of intervening third parties, such as surviving joint tenants. Thus, the grantee’s acceptance of the deed after the grantor’s death does not relate back to defeat the right of survivorship. Here, the grandson did not accept the deed until after the landowner’s death. In the meantime and because of that death, the friend’s right to the whole property had accrued to her as the surviving joint tenant.

74
Q

Confrontation clause – what amounts to a testimonial statements

A homeowner returned home from work one day to find a robber in her living room. After a brief physical altercation, the homeowner ran to a bedroom, hid in a closet, and called 911 on her cell phone. Police officers arrived in less than two minutes and were able to apprehend the robber as he tried to run out the front door. Once they made sure he was locked in the police car, one of the officers went to speak with the homeowner about what had happened. She was still crying and shaking when the officer found her, and she said, “Thank you for catching him! He punched me in the head as I was running away!” The robber was charged with robbery and assault. Traumatized, the homeowner left the country and cannot be traced, despite the efforts of the prosecutor. The prosecutor intends to call the officer to testify as to the homeowner’s statement.
Should the court allow the officer’s testimony?

A

The testimony should not be admitted. Under the Confrontation Clause, an accused has the right to be confronted by the witnesses against him. A hearsay statement will not be admitted—even if it falls within a hearsay exception—when: (i) the statement is offered against the accused in a criminal case; (ii) the declarant is unavailable; (iii) the statement was testimonial in nature; and (iv) the accused had no opportunity to cross-examine the declarant’s “testimonial” statement prior to trial. The Supreme Court has established that if the primary purpose of police interrogation is to enable the police to help in an ongoing emergency, statements made in the course of the interrogation are nontestimonial. When the primary purpose of the interrogation is to establish or prove past events potentially relevant to a later criminal prosecution, statements are testimonial. Here, the homeowner’s statement was hearsay because it was made out of court and is being offered for its truth—that the robber punched her in the head. It appears that the emergency had already resolved by the time the statement was made. Although only a few minutes had passed since the physical altercation and the homeowner was still upset, the robber no longer posed any danger because he was locked in the police car and the homeowner was aware of this (“Thank you for catching him!”). Therefore, the homeowner’s statements to the officer were testimonial. Because the homeowner is unavailable to testify at trial and the robber has had no opportunity to cross-examine the statements, admitting them at trial through the testimony of the officer would violate the Confrontation Clause.

75
Q

Direct or circumstantial evidence?

A plaintiff sued a defendant and his employer for personal injuries. The plaintiff claimed that she was struck on the head by a wrench dropped by the defendant from a high scaffold, on which the defendant was working in the course of a construction project. To prove that it was the defendant who dropped the wrench, the plaintiff offers the wrench itself as evidence: The wrench bears the brand name “Craftsman” on the handle, and other evidence shows that the wrenches used by the defendant on the job are “Craftsman” brand wrenches.

Is the wrench admissible?

A

he word “Craftsman” is not hearsay, and the wrench is relevant circumstantial evidence on the issue of whether the defendant dropped the wrench that struck the plaintiff. Evidence is relevant if it tends to make the existence of any fact of consequence to the action more probable than it would be without the evidence. [Fed. R. Evid. 401] If the defendant uses “Craftsman” wrenches on the job, and the wrench that struck the plaintiff bears the brand name “Craftsman,” it is more probable than would otherwise be the case that the wrench that struck the plaintiff was dropped by the defendant. Thus, the wrench is relevant to prove that the defendant dropped the wrench. The wrench is circumstantial, rather than direct, evidence because a fact about it is being proved as a basis for an inference that another fact is true; i.e., the fact that the wrench bears the name “Craftsman” is proved to form a basis for inferring that the defendant dropped the wrench. Direct evidence is offered to prove a fact about the object as an end in itself. Here, the wrench bearing the name “Craftsman” is not being offered as a means of proving, e.g., that the wrench is in fact a “Craftsman” brand. Consequently, the wrench constitutes circumstantial evidence. In addition, when offered for the stated purpose, the wrench is not hearsay. Hearsay is a statement, other than one made by the declarant while testifying at the trial or hearing, offered in evidence to prove the truth of the matter asserted. [Fed. R. Evid. 801(c)] The wrench is not being offered to prove the truth of the matter asserted (i.e., that the wrench is actually a “Craftsman”). It is of no significance whether the wrench being offered is a genuine “Craftsman.” What is important is that it bears the same name as those wrenches used by the defendant on the job. Thus, introduction of the wrench into evidence will not violate the rule against hearsay.

76
Q

To better reflect the age range of its citizens, a city council passed an ordinance providing that no city employee could be hired or promoted unless that employee had reached the age of 55. A 25-year-old city mechanic in the lower pay classification had recently finished first on a promotional exam for the senior mechanic position, which would entitle him to a substantial increase in pay. His supervisor told him that, under instructions from the head of the city administrative office, a 56-year-old city mechanic who scored lower on the exam would be promoted to the position instead.

If the mechanic brings an appropriate action in federal court to challenge the ordinance, which party would bear the burden of proof?

A The mechanic, to prove that there is no compelling state interest furthered by the challenged ordinance.

B The mechanic, to prove that the challenged ordinance is clearly arbitrary and irrational.

C The city, to prove that its ordinance is necessary to further a compelling state interest.

D The city, to prove that there was a rational basis for enacting the challenged ordinance.

A

B…look back at your outline

77
Q

Note on “indispensable parties”

A

The court should deny the truck driver’s motion because the wife is not “needed for just adjudication.” When deciding whether an absentee party is indispensable, the court considers a number of factors. However, the Supreme Court has held that a joint tortfeasor subject to joint and several liability is not a person needed for just adjudication. [Temple v. Synthes Corp., 498 U.S. 5 (1990)] Because the wife here is a joint tortfeasor subject to joint and several liability, she is not an indispensable party in this case.

78
Q

A landlord entered into a written lease of a bakery for a term of 25 years with a baker. The parties agreed to a right of first refusal if the bakery was offered for sale during the term of the lease. The lease also permitted assignments and subleases on notice to the landlord. Three years later, the baker retired and, after notifying the landlord, transferred the lease to a chocolatier. Twenty-one years later, the landlord entered into a contract with a buyer for the sale of the bakery for $100,000. The landlord had informed the buyer of the lease but had forgotten about the right of first refusal. When the chocolatier learned of the sale to the buyer, she informed both the landlord and the buyer that she wanted to exercise her option and was prepared to purchase the bakery for the contract price. The jurisdiction’s Rule Against Perpetuities is unmodified by statute.

Can the chocolatier enforce the option?

A Yes, because an option held by a tenant on leased property cannot be separated from the leasehold interest.

B Yes, because the option touches and concerns the leasehold estate.

C No, because the transfer to the chocolatier made the option void under the Rule Against Perpetuities.

D No, because the option was not specifically included when the lease was transferred to the chocolatier.

A

Note that no notice is required for benefit (of covenant or equitable servitude to run with the land)

B) The chocolatier can enforce the option to purchase because it is a covenant that runs with the land. When a tenant makes a complete transfer of the entire remaining term of his leasehold interest, it constitutes an assignment. The assignee and the landlord are then in privity of estate, and each is liable to the other on all covenants in the lease that run with the land. The covenant runs with the land if the original parties so intend and the covenant “touches and concerns” the leased land, i.e., burdens the landlord and benefits the tenant with respect to their interests in the property. Here, the transfer of the lease to the chocolatier was an assignment, making all covenants in the lease that run with the land enforceable by the assignee. The right of first refusal burdens the landlord’s power of alienation over the bakery, and there is nothing to indicate that the parties intended the option to be personal to the baker. Hence, the chocolatier can enforce the option and purchase the property.

79
Q

A landowner owned two adjoining parcels of land. The landowner sold the western parcel to a buyer by a deed that contained the following clause: “Grantee promises for himself, his heirs, successors, and assigns to not erect a structure over two stories on the land.” The buyer recorded the deed and built a two-story house on the property and lived there for 30 years, after which he sold the land to a movie star by a deed that did not contain the structure height restriction. The movie star decided to tear down the existing house on the land and erect a three-story house. Her neighbor, who purchased the eastern parcel from the landowner 10 years earlier, discovers that the movie star’s house will be larger than his and files suit to enforce the covenant.

Who will prevail?

A The neighbor, because the restrictive covenant runs with the land.

B The neighbor, because privity is not required to enforce an equitable servitude.

C The movie star, because her deed did not contain the restrictive covenant.

D The movie star, because she had no notice of the structure height restriction.

A

Touches and concerns land issue
The neighbor will prevail because the restrictive covenant runs with the neighbor’s land, which is benefited, and with the movie star’s land, which is burdened. A covenant at law will run with the land and be enforceable against subsequent grantees if: (i) the contracting parties intended it to run; (ii) there is privity of estate between the original promisor and promisee (horizontal privity), as well as between the promisor and his successor (vertical privity); (iii) the covenant touches and concerns the property; and (iv) the burdened party has notice of the covenant. Here, the use of the words “heirs, successors, and assigns” in the covenant shows the intent for the covenant to run. The original parties were in horizontal privity because at the time the buyer entered into the covenant, he and the landowner shared an interest in the land independent of the covenant—as grantor and grantee. Because the neighbor and the movie star hold the entire interest in the parcels held by the landowner and the buyer, respectively, there is vertical privity. The covenant touches and concerns the land because it diminishes the movie star’s rights in connection with her enjoyment of the western parcel. Finally, the movie star has notice of the covenant because it was recorded in her chain of title (in the landowner-buyer deed). Thus, the covenant runs with the land. (B) is incorrect because although privity is not required to enforce an equitable servitude, the covenant here runs with the land at law. Moreover, privity does exist under these facts, as explained above.

80
Q

Can you include a notorized document by a public dude who is now available saying that something is not in a public record?

A

YES The document should be admitted. Related to the exception to the hearsay rule for public records and other official writings, Federal Rule of Evidence 803(10) provides that evidence in the form of a certification or testimony from the custodian of public records that she has diligently searched and failed to find a record is admissible to prove that a matter was not recorded, or, inferentially, that a matter did not occur. Here, the defendant’s status as a licensed physical therapist would normally be revealed in the records of the department. The document here at issue is admissible, under the foregoing hearsay exception, as a means of proving that the defendant is in fact not licensed

81
Q

“Bob is extremely caution and invariably follows traffic rules” character or habit evidence?

A

A) is incorrect. Under Federal Rule 406, evidence of a person’s habit may be admitted to prove that on a particular occasion the person acted in accordance with the habit. Habit describes one’s regular response to a specific set of circumstances, while character describes one’s disposition in respect to general traits. Since habits are more specific and particularized, evidence of habit is relevant and can be introduced in circumstances when it is not permissible to introduce evidence of character. Here, however, the friend’s testimony is character evidence, not habit evidence. The fact that the pedestrian is cautious and obeys traffic laws is not nearly specific enough to qualify as habit evidence—despite the use of the term “invariably

82
Q

Interesting question:

A boy planned to rob the local currency exchange, but he needed a getaway car and somebody to be a lookout. He asked his sister, who immediately refused and warned him against carrying it out. The boy showed his sister a gun, threatening to shoot her if she did not go along with the plan. The sister consented to help, and the pair left for the currency exchange, with the sister driving. When they arrived, the sister left the car running so that she could get away and contact the police while her brother was inside, but the boy ordered her to go inside with him. During the robbery attempt, the clerk pulled out a gun, and the boy and the clerk shot and killed each other.

If the sister is charged with the killing of the store clerk, should the jury find her criminally liable if it accepts the sister’s version of the facts?

A Yes, because duress is not a defense to murder.

B Yes, because the clerk’s death occurred in furtherance of and during the perpetration of a robbery in which the sister was participating.

C No, because the sister lacked the requisite intent.

D No, because the sister withdrew from the conspiracy to commit robbery and thus is not liable for any acts committed in furtherance of the conspiracy.

A

The sister is not criminally liable for the store clerk’s death. A defendant may, by virtue of her participation in a conspiracy, be liable for the crimes of all other conspirators if the crimes were committed in furtherance of the objectives of the conspiracy and were a natural and probable consequence of the conspiracy (i.e., were foreseeable). Furthermore, liability for felony murder would attach if the killing were committed during the course of a felony. Thus, the sister would be criminally liable for the killing of the clerk if she were a co-conspirator or if she were a principal to the underlying felony (the robbery). However, under the facts, the sister is neither because she lacked the requisite intent. Given the threat to her life, the sister did not freely enter into a conspiracy to commit robbery. (A) is incorrect because, while duress is not a defense to murder, it is a defense to the underlying felony, and thus is a defense to the felony murder charge. Here, the sister was acting under duress during the robbery, and thus she has a defense to the robbery charge.

83
Q

The plaintiff was exiting from a parking garage owned and operated by the city when he discovered that the exit ramp was blocked by construction barricades and a pile of broken-up concrete. No workers or detour signs were around and the plaintiff was in a hurry, so he backed up and drove down an entrance ramp that was clearly marked as such. As he came around a corner, his car was broadsided by a pickup truck. The plaintiff was seriously injured in the collision. A statute in the jurisdiction requires drivers to obey all traffic directional markings in both public and private parking lots and garages. The jurisdiction retains governmental immunity for municipalities.

If the plaintiff brings a lawsuit against the city to recover for his injuries, which of the following facts will be LEAST helpful in the city’s defense?

A The plaintiff was aware that another exit on the other side of the garage was open.

B The construction workers responsible for blocking off the exit ramp were employees of an independent contractor rather than the city.

C The city does not collect fees or make a profit in the operation of the garage.

D The pickup truck driver could have avoided the plaintiff but recognized him as an old enemy and deliberately ran into him.

A

The fact least helpful to the city’s defense of the plaintiff’s lawsuit is the identity of the workers who blocked the exit ramp. Under vicarious liability rules, a principal will be liable for the tortious acts of an independent contractor if the duty is nondelegable on public policy grounds; included is the duty of a possessor of land to keep its premises safe for its invitees. If the workers were negligent in leaving the ramp blocked without providing another means of exiting, the fact that they were not city employees would not absolve the city of liability; hence, their identity would be of no help to the city’s defense

84
Q

Which of the following is character evidence, rather than habit evidence?
response - correct

A “Ben is always in a hurry.”

B “Ben always wears his seatbelt.”

C “Ben goes to church every Sunday.”

D “Ben goes to the pub every Friday night at 7 p.m.”

A

A correct

85
Q

Under the Confrontation Clause, prior testimonial evidence may not be admitted unless: (i) the declarant is unavailable; and (ii) defendant had an opportunity to cross-examine the declarant at the time the statement was made. Which of the following is considered “nontestimonial” evidence for purposes of the Confrontation Clause?

A Affidavits that summarize the results of a fingerprint test.

B Statements made by a robbery victim to a police officer about the details of the robbery.

C Statements made about the identity of the perpetrators during a 911 call in the course of a home invasion.

D Statements made by a victim of a theft to a police officer about the physical features of the suspect.

A

C

86
Q

Plaintiff wants to introduce a statement made in a prior case by a now-unavailable witness. Which of the following is NOT a requirement of the former testimony exception to the hearsay rule?
response - correct

A The former testimony must have been given under oath or sworn affirmation.

B The subject matter in the current case and the prior case are the same.

C The parties in the current case must be the same as in the prior case.

D Defendant (or her predecessor in interest) must have had the opportunity to question the declarant about this statement in the prior case.

A

C

87
Q

“I intend to go to Hawaii next week” State of mind exception or present sense impression?

A

“I intend to go to Hawaii next week” falls under the state of mind hearsay exception. Declarations of existing state of mind are admissible (i) when declarant’s state of mind is directly in issue, or (ii) if they are declarations of intent offered to show subsequent acts of the declarant. This statement would be offered to show the subsequent act of the declarant; i.e., that he actually went to Hawaii. A statement of present sense impression describes an event or condition happening contemporaneously with or immediately prior to the statement. “I intend to go to Hawaii next week” is not a present sense impression because it concerns the declarant’s future act. The excited utterance exception applies to statements made by a declarant during or soon after a startling event and while under the stress of the event. Here, there is no indication that the plaintiff made his statement while under the stress of a startling event. The exception for declarations of present bodily condition applies to statements regarding physical symptoms, e.g., pain. The declarant’s statement that he intended to go to Hawaii does not fall under this exception.